Finding the general form of a Struve Function

Click For Summary
The discussion revolves around identifying the general form of a Struve function from a given series. The series features alternating terms with odd powers of x in the numerator and a product of squared odd integers in the denominator. The user initially struggled to find a suitable expression for the denominator but later realized that using (2k+1)!^2 could be the solution. They also noted the importance of using double factorials in Mathematica for accurate calculations. The conversation highlights the challenges of series representation and the nuances of computational tools.
eclayj
Messages
20
Reaction score
0
I am given the following series which the problem states is known as a Struve function:

x -x^3/(1^2 3^2)+x^5/(1^2 3^2 5^2)-x^7/(1^2 3^2 5^2 7^2) + ...

I am trying to figure out the expression of the series. I have thought about this for some time. I am sure I am missing the obvious. It looks a little like the expansion of Sin[x] in powers of x. Anyway, I have worked at the numerator as follows:

-1^k x^(2k+1)

I cannot figure out for the life of me an expression that will give the denominator of the terms in the series (odd numbers squared and multiplied, i.e. the first term's denominator is one squared, then the next term has the denominator one squared times three squared, etc.).

If anyone could help me think about this it would be greatly appreciated. Thanks so much! I have attached a .nb file to run in Mathematica if it helps. It is the second question.

-EJ
 

Attachments

Physics news on Phys.org
What about (2k+1)!^2?
 
Wow. Pretty obvious. Maybe I shouldn't be doing so late at night. Thank you very much!
 
Also didn't realize that you need two factorial symbols "!" rather than the usual one symbol for Mathematica to recognize and properly calculate the equation.
 
Question: A clock's minute hand has length 4 and its hour hand has length 3. What is the distance between the tips at the moment when it is increasing most rapidly?(Putnam Exam Question) Answer: Making assumption that both the hands moves at constant angular velocities, the answer is ## \sqrt{7} .## But don't you think this assumption is somewhat doubtful and wrong?

Similar threads

  • · Replies 7 ·
Replies
7
Views
1K
  • · Replies 1 ·
Replies
1
Views
2K
  • · Replies 2 ·
Replies
2
Views
858
Replies
2
Views
1K
  • · Replies 14 ·
Replies
14
Views
2K
  • · Replies 40 ·
2
Replies
40
Views
4K
Replies
2
Views
2K
  • · Replies 17 ·
Replies
17
Views
3K
  • · Replies 7 ·
Replies
7
Views
1K
  • · Replies 3 ·
Replies
3
Views
2K